LSAT Community

Help - Could you please explain the reason why (E) shouldn't be the answer? Thanks,
Batman July 10, 2014
  • Exam Questions
  • Reading Comp
  • Humanities
  • June 1997 - Sec 1 - Q8
Help - How could both approach be complement each other according to the passage? I don't understand tha...
Batman July 10, 2014
  • Exam Questions
  • Reading Comp
  • Humanities
  • June 1997 - Sec 1 - Q2
Why is this B and not D? - Will you please break this problem down for me.
OffshoreSuge July 10, 2014
  • Exam Questions
  • Logical Reasoning
  • Must Be True
  • October 2011 - Sec 3 - Q18
Don't quite understand - Could you explain why the correct answer is B? Thank you!
TTabriz July 9, 2014
  • Exam Questions
  • Logical Reasoning
  • Argument Structure
Why? - I got the right answer but that was because it seemed like the best choice. The question asks, t...
Derek July 8, 2014
  • Exam Questions
  • Logic Games
  • Group Games
I am still not understanding this question - The question stem asks what Must be True, so what has to be true given the information. Answer B ...
Derek July 8, 2014
  • Exam Questions
  • Logic Games
  • Group Games
Explain - Is it because since it is not legally impermissible it is legally permissible, making a legally p...
Jborj653 July 8, 2014
  • Exam Questions
  • Logical Reasoning
  • Cannot Be True
Explain - Yeah I was confused on this one as well. I ended up going with e but I knew the connection betwee...
Jborj653 July 8, 2014
  • Exam Questions
  • Logical Reasoning
  • Cannot Be True
Why A? - First of all it says "fan members" not "family members" but I think after you read it a couple ti...
Derek July 8, 2014
  • Exam Questions
  • Logical Reasoning
  • Cannot Be True
Assistance please - Hey guys, Is it possible to see a set-up for this game?
Titan July 7, 2014
  • Exam Questions
  • Logic Games
  • The Lost Boys